Difference between revisions of "1981 AHSME Problems/Problem 13"

(Problem)
(Solution)
 
Line 5: Line 5:
  
 
==Solution==
 
==Solution==
What we are trying to solve is <math>log_{0.9}^{0.1}=n</math>. This turns into <math>\frac{log{0.1}}{log{0.9}}=\frac{-1}{log{9}-1}=n</math> We know that <math>log_{10}^{3}=0.466</math>, thus by log rules we have <math>2log_{10}^{3}=log_{10}^{9}=2*0.477=0.954</math>, thus <math>n=\frac{1}{.046}>\ans{21}</math>
+
What we are trying to solve is <math>log_{0.9}^{0.1}=n</math>. This turns into <math>\frac{log{0.1}}{log{0.9}}=\frac{-1}{log{9}-1}=n</math> We know that <math>log_{10}^{3}=0.466</math>, thus by log rules we have <math>2log_{10}^{3}=log_{10}^{9}=2*0.477=0.954</math>, thus <math>n=\frac{1}{.046} > 21</math>, and our answer is <math>\boxed{(B) 22}</math>
  
 
-edited by Maxxie
 
-edited by Maxxie

Latest revision as of 00:12, 8 September 2024

Problem

Suppose that at the end of any year, a unit of money has lost 10% of the value it had at the beginning of that year. Find the smallest integer $n$ such that after $n$ years, the money will have lost at least $90\%$ of its value (To the nearest thousandth $log_{10}^{3}=0.477$).

$\textbf{(A)}\ 14\qquad\textbf{(B)}\ 16\qquad\textbf{(C)}\ 18\qquad\textbf{(D)}\ 20\qquad\textbf{(E)}\ 22$

Solution

What we are trying to solve is $log_{0.9}^{0.1}=n$. This turns into $\frac{log{0.1}}{log{0.9}}=\frac{-1}{log{9}-1}=n$ We know that $log_{10}^{3}=0.466$, thus by log rules we have $2log_{10}^{3}=log_{10}^{9}=2*0.477=0.954$, thus $n=\frac{1}{.046} > 21$, and our answer is $\boxed{(B) 22}$

-edited by Maxxie